Last visit was: 25 Apr 2024, 08:17 It is currently 25 Apr 2024, 08:17

Close
GMAT Club Daily Prep
Thank you for using the timer - this advanced tool can estimate your performance and suggest more practice questions. We have subscribed you to Daily Prep Questions via email.

Customized
for You

we will pick new questions that match your level based on your Timer History

Track
Your Progress

every week, we’ll send you an estimated GMAT score based on your performance

Practice
Pays

we will pick new questions that match your level based on your Timer History
Not interested in getting valuable practice questions and articles delivered to your email? No problem, unsubscribe here.
Close
Request Expert Reply
Confirm Cancel
SORT BY:
Date
Tags:
Difficulty: 605-655 Levelx   Weakenx               
Show Tags
Hide Tags
avatar
Intern
Intern
Joined: 26 Jan 2008
Posts: 14
Own Kudos [?]: 160 [145]
Given Kudos: 0
Send PM
Most Helpful Reply
CEO
CEO
Joined: 24 Jul 2011
Status: World Rank #4 MBA Admissions Consultant
Posts: 3187
Own Kudos [?]: 1585 [30]
Given Kudos: 33
GMAT 1: 780 Q51 V48
GRE 1: Q170 V170
Send PM
User avatar
Retired Moderator
Joined: 16 Jun 2012
Posts: 871
Own Kudos [?]: 8554 [23]
Given Kudos: 123
Location: United States
Send PM
General Discussion
User avatar
Manager
Manager
Joined: 06 Apr 2008
Posts: 158
Own Kudos [?]: 487 [1]
Given Kudos: 1
Send PM
Re: The proposal to hire ten new police officers in Middletown is quite fo [#permalink]
1
Kudos
ttwang56 wrote:
The proposal to hire ten new police officers in Middletown is quite foolish. There is sufficient funding to pay the salaries of the new officers, but not the salaries of additional court and prison employees to process the increased caseload of arrests and convictions that new officers usually generate. Which of the following, if true, will most seriously weaken the conclusion drawn above?
a. Studies had shown that an increase in city’s place force does not necessarily reduce crime
b. When one major city increased its police force by 19% last year, there were 40% more arrests and 13% more convictions
c. If funding for the new police officers’ salaries is approved, support for other city services will have to be reduced during the next fiscal year
d. In most US cities, not all arrests result in convictions, and not all convictions result in prison terms
e. Middletown’s ratio of police officers to citizens has reached a level at which an increase in the number of officers will have a deterrent effect on crime


IMO E)

If crime will be less then number of convictions/arrests will be less as well
avatar
Intern
Intern
Joined: 26 Jan 2008
Posts: 14
Own Kudos [?]: 160 [0]
Given Kudos: 0
Send PM
Re: The proposal to hire ten new police officers in Middletown is quite fo [#permalink]
doesn't that support the argument though?
User avatar
Senior Manager
Senior Manager
Joined: 20 Sep 2006
Posts: 477
Own Kudos [?]: 299 [6]
Given Kudos: 7
Send PM
Re: The proposal to hire ten new police officers in Middletown is quite fo [#permalink]
5
Kudos
1
Bookmarks
ttwang56 wrote:
doesn't that support the argument though?



Nope ... Its is weakening the argument here the flow:

More police --> Less Crime --> less arrests --> Less employes in court and prison to process the caseload of arrests. hence the proposal is not bad et all.
User avatar
Manager
Manager
Joined: 11 Jun 2011
Status:Dream big, work hard, and drink gallons of beer!
Posts: 98
Own Kudos [?]: 229 [2]
Given Kudos: 33
Concentration: Finance, General Management
GMAT Date: 10-01-2011
WE:Web Development (Consulting)
Send PM
Re: The proposal to hire ten new police officers in Middletown is quite fo [#permalink]
2
Kudos
E is Right. If crime is reduced, salaries for addition court and prison employees are not needed, so the decision to hire new officers is not really foolish.
Verbal Forum Moderator
Joined: 08 Dec 2013
Status:Greatness begins beyond your comfort zone
Posts: 2101
Own Kudos [?]: 8809 [0]
Given Kudos: 171
Location: India
Concentration: General Management, Strategy
GPA: 3.2
WE:Information Technology (Consulting)
Send PM
Re: The proposal to hire ten new police officers in Middletown is quite fo [#permalink]
The proposal to hire ten new police officers in Middletown is quite foolish. There is sufficient funding to pay the salaries of the new officers, but not the salaries of additional court and prison employees to process the increased caseload of arrests and convictions that new officers usually generate.

Type - weaken
Boil it down - The proposal to hire 10 new police officers is foolish .
- Although there is sufficient funding for new officer salaries , but not the salaries of additional court and prison employees to process increase caseload of arrests

Pre-Thinking - Some reason due to which the caseload of arrests and convictions won't increase

Which of the following, if true, will most seriously weaken the conclusion drawn above?
a. Studies had shown that an increase in city’s place force does not necessarily reduce crime - Incorrect
b. When one major city increased its police force by 19% last year, there were 40% more arrests and 13% more convictions - Incorrect - we don't know 10 is what percentage of workforce but we can infer that caseload of arrests and convictions will increase
c. If funding for the new police officers’ salaries is approved, support for other city services will have to be reduced during the next fiscal year - Irrelevant
d. In most US cities, not all arrests result in convictions, and not all convictions result in prison terms - Incorrect as convictions and prison terms still might increase
e. Middletown’s ratio of police officers to citizens has reached a level at which an increase in the number of officers will have a deterrent effect on crime . Correct -
The new officers will have a deterrent effect on crime . So , no additional expenditure on support services will be needed

Answer E
Manager
Manager
Joined: 26 Feb 2015
Posts: 61
Own Kudos [?]: 195 [0]
Given Kudos: 109
GPA: 3.92
Send PM
Re: The proposal to hire ten new police officers in Middletown is quite fo [#permalink]
When potential answers use phrases like "most" or "almost all", it is usually a sign of a trap answer.
Manager
Manager
Joined: 04 Aug 2015
Posts: 60
Own Kudos [?]: 78 [1]
Given Kudos: 36
Location: India
Concentration: Leadership, Technology
GMAT 1: 700 Q50 V35
GPA: 3.39
Send PM
Re: The proposal to hire ten new police officers in Middletown is quite fo [#permalink]
1
Kudos
The proposal to hire ten new police officers in Middletown is quite foolish. There is sufficient funding to pay the salaries of the new officers, but not the salaries of additional court and prison employees to process the increased caseload of arrests and convictions that new officers usually generate.

Which of the following, if true, will most seriously weaken the conclusion drawn above?

Structure: More cops --(1)--> More arrests and increased caseload --(2)--> Additional court and prison employees required and more salaries to be paid

To Weaken:
Linkage 1: More cops might not lead to increased number of arrests. Maybe crimes won't take place at the first place because of the increased number of cops.
Linkage 2: If the current workforce is able to handle the increased caseload then there won't be a requirement for an additional workforce.


a. Studies had shown that an increase in city’s place force does not necessarily reduce crime
Two situations stem from this point: (a) Crime rate remains same - in such a case no additional workforce is required. (b) Crime rate increases - in such a case an additional workforce might be required. Since we have both the possibilities, so not a strong option.
b. When one major city increased its police force by 19% last year, there were 40% more arrests and 13% more convictions
This point strengthens the Linkage 1 whereas we are here to weaken.
c. If funding for the new police officers’ salaries is approved, support for other city services will have to be reduced during the next fiscal year
Other city services - Out of Scope.
d. In most US cities, not all arrests result in convictions, and not all convictions result in prison terms
Let's imagine two scenarios:
SC1: [Before the new cops are hired] Arrests: 100 Convictions: 99 (not all) Prison Terms: 98 (not all).
SC2: [After the new cops are hired] Arrests: 150 Convictions: 149 (not all) Prison Terms: 148 (not all).
Even though the condition of 'not all' is satisfied across the scenarios, we might have a case which strengthens the argument.

e. Middletown’s ratio of police officers to citizens has reached a level at which an increase in the number of officers will have a deterrent effect on crime
This point hits the Linkage 1. An increase in the number of cops WILL bring down the crime rate and, thus, won't lead to an increased workload.

Thus, I chose option E.
Manager
Manager
Joined: 15 Jun 2016
Posts: 91
Own Kudos [?]: 23 [0]
Given Kudos: 741
Send PM
Re: The proposal to hire ten new police officers in Middletown is quite fo [#permalink]
I ma confused in option A and E, could you please explain?
CR Moderator
Joined: 14 Dec 2013
Posts: 2413
Own Kudos [?]: 15266 [1]
Given Kudos: 26
Location: Germany
Schools:
GMAT 1: 780 Q50 V47
WE:Corporate Finance (Pharmaceuticals and Biotech)
Send PM
Re: The proposal to hire ten new police officers in Middletown is quite fo [#permalink]
1
Bookmarks
Expert Reply
VKat wrote:
I ma confused in option A and E, could you please explain?


A and E states the opposite things.
A: increase in police force ---> no decrease in crime. Thus the argument is strengthened.
E: increase in police force ---> decrease in crime. Thus the argument is weakened.
Manager
Manager
Joined: 08 Feb 2016
Posts: 53
Own Kudos [?]: 60 [1]
Given Kudos: 25
Location: India
Concentration: Technology
Schools: AGSM '20 (A)
GMAT 1: 650 Q49 V30
GPA: 4
Send PM
Re: The proposal to hire ten new police officers in Middletown is quite fo [#permalink]
1
Kudos
StoicBread wrote:
When potential answers use phrases like "most" or "almost all", it is usually a sign of a trap answer.


I disagree with this.
'Most' is greater than 50%. I can say it is 51% to 99%. Option D, IMHO, is not wrong because it uses 'most'. Option D is wrong because it uses 'not all'. 'Not all' may range from 0 % - 99%.
Not all may mean 0% arrests result In conviction, i.e. no arrests lead to conviction. If this is the case then this option definitely weakens. However, if it means 99% arrests result in conviction, then the argument is strengthened, not weakened. Thus D is ambiguous because it uses ‘not all’ which has a variable effect on the argument.

Similarly, 'almost all' is close to 100%. It's quite different from 'not all'.

sayantanc2k - please correct me if I am wrong.
Manager
Manager
Joined: 05 Jan 2016
Status:Final Call! Will Achieve Target ANyHow This Tym! :)
Posts: 70
Own Kudos [?]: 153 [0]
Given Kudos: 135
Location: India
GMAT 1: 620 Q49 V25
GPA: 3.8
Send PM
Re: The proposal to hire ten new police officers in Middletown is quite fo [#permalink]
TomB wrote:
The proposal to hire ten new police officers in Middletown is quite foolish. There is sufficient funding to pay the salaries of the new officers, but not the salaries of additional court and prison employees to process the increased caseload of arrests and convictions that new officers usually generate. Which of the following, if true, will most seriously weaken the conclusion drawn above?
a. Studies had shown that an increase in city’s police force does not necessarily reduce crime
b. When one major city increased its police force by 19% last year, there were 40% more arrests and 13% more convictions
c. If funding for the new police officers’ salaries is approved, support for other city services will have to be reduced during the next fiscal year
d. In most US cities, not all arrests result in convictions, and not all convictions result in prison terms
e. Middletown’s ratio of police officers to citizens has reached a level at which an increase in the number of officers will have a deterrent effect on crime

can anybody explain option A. I cannot ruled out this option


Hi TomB,

I am happy to respond. :)

A. Studies had shown that an increase in city’s police force does not necessarily reduce crime.
So, INCREASE IN CITY'S POLICE FORCE --> NOT NECESSARILY REDUCE CRIME. Thus, more crime so the arrests and convictions that new officers usually generate will even be more. So, the proposal to hire ten new police officers in Middle town is not quite foolish. It strengthens. Therefore, incorrect.
Manager
Manager
Joined: 05 Jan 2016
Status:Final Call! Will Achieve Target ANyHow This Tym! :)
Posts: 70
Own Kudos [?]: 153 [0]
Given Kudos: 135
Location: India
GMAT 1: 620 Q49 V25
GPA: 3.8
Send PM
Re: The proposal to hire ten new police officers in Middletown is quite fo [#permalink]
Explanation in detail -

The passage says that hiring new officers usually brings new court expenses, but according to choice E hiring new officers in Middletown will lead to a reduction in crime and thus, perhaps, a reduction in court and prison expenses. Therefore, choice E weakens the conclusion drawn and is the best answer.

Three of the other choices tend to support claims made in the passage;
choice A suggests that arrests will increase, therefore, strengthens,
choice B says that in one city arrests did increase, therefore, strengthens,
choice C confirms the scarcity of funds, therefore, strengthens that the proposal to hire ten new police officers in Middletown is NOT quite foolish.
Choice D is irrelevant; it merely states the obvious about rates of arrest, conviction, and imprisonment.
Director
Director
Joined: 20 Dec 2015
Status:Learning
Posts: 876
Own Kudos [?]: 566 [0]
Given Kudos: 755
Location: India
Concentration: Operations, Marketing
GMAT 1: 670 Q48 V36
GRE 1: Q157 V157
GPA: 3.4
WE:Engineering (Manufacturing)
Send PM
Re: The proposal to hire ten new police officers in Middletown is quite fo [#permalink]
ttwang56 wrote:
The proposal to hire ten new police officers in Middletown is quite foolish. There is sufficient funding to pay the salaries of the new officers, but not the salaries of additional court and prison employees to process the increased caseload of arrests and convictions that new officers usually generate. Which of the following, if true, will most seriously weaken the conclusion drawn above?
a. Studies had shown that an increase in city’s place force does not necessarily reduce crime
b. When one major city increased its police force by 19% last year, there were 40% more arrests and 13% more convictions
c. If funding for the new police officers’ salaries is approved, support for other city services will have to be reduced during the next fiscal year
d. In most US cities, not all arrests result in convictions, and not all convictions result in prison terms
e. Middletown’s ratio of police officers to citizens has reached a level at which an increase in the number of officers will have a deterrent effect on crime


The answer is E

The reasoning is that if there are sufficient police officers then people would would not commit crime because they would be caught .
The increase in the number of police officers will thus increase vigilance and increase in number will act as deterrent .
Verbal Forum Moderator
Joined: 08 Dec 2013
Status:Greatness begins beyond your comfort zone
Posts: 2101
Own Kudos [?]: 8809 [0]
Given Kudos: 171
Location: India
Concentration: General Management, Strategy
GPA: 3.2
WE:Information Technology (Consulting)
Send PM
Re: The proposal to hire ten new police officers in Middletown is quite fo [#permalink]
The proposal to hire ten new police officers in Middletown is quite foolish. There is sufficient funding to pay the salaries of the new officers, but not the salaries of additional court and prison employees to process the increased caseload of arrests and convictions that new officers usually generate.

Type - weaken

a. Studies had shown that an increase in city’s place force does not necessarily reduce crime - Incorrect - as it strengthens
b. When one major city increased its police force by 19% last year, there were 40% more arrests and 13% more convictions - Incorrect as it strengthens the argument
c. If funding for the new police officers’ salaries is approved, support for other city services will have to be reduced during the next fiscal year. - Irrelevant - we are not bothered how the funding is managed
d. In most US cities, not all arrests result in convictions, and not all convictions result in prison terms - Irrelevant
e. Middletown’s ratio of police officers to citizens has reached a level at which an increase in the number of officers will have a deterrent effect on crime - weakens - If this is true , then more police will result in lesser crime .
Intern
Intern
Joined: 11 Dec 2019
Posts: 15
Own Kudos [?]: 14 [0]
Given Kudos: 17
Location: Uzbekistan
GMAT 1: 710 Q49 V37
Send PM
Re: The proposal to hire ten new police officers in Middletown is quite fo [#permalink]
The question stem already mentions that new officers will generate new caseloads of arrests. What happens when they generate more arrests? -> not decrease in crime? All i mean is that it is aready implied that crime level will decrease after hiring new officers, hence E just restates an implied fact.
Besides, the question is quite misleading on cause and effect:
- by OA E it implies that decrease in crime -> decrease in arrests and caseloads. But decreasing crime itself requires arrests and caseloads and will eventually lead to higher imprisonment. It is just a question of time - when will the officers be able to reach minimum crime level

I hope it is not an OG question
Manager
Manager
Joined: 15 Jul 2014
Posts: 92
Own Kudos [?]: 103 [0]
Given Kudos: 233
Location: India
Concentration: Marketing, Technology
Send PM
Re: The proposal to hire ten new police officers in Middletown is quite fo [#permalink]
BoburMirzo26 wrote:
The question stem already mentions that new officers will generate new caseloads of arrests. What happens when they generate more arrests? -> not decrease in crime? All i mean is that it is aready implied that crime level will decrease after hiring new officers, hence E just restates an implied fact.
Besides, the question is quite misleading on cause and effect:
- by OA E it implies that decrease in crime -> decrease in arrests and caseloads. But decreasing crime itself requires arrests and caseloads and will eventually lead to higher imprisonment. It is just a question of time - when will the officers be able to reach minimum crime level

I hope it is not an OG question


I am not sure whether this is an OG question, but I certainly can point out that increase in arrests does not result in reduction of crime. This is because, the argument says that only increase in in crime will happen and that this increase will burden other departments.
Now, option E says increase in arrests will result in reduction of crimes. There will be a saturation point beyond which the crime rates will get reduced and the city had reached the point.
VP
VP
Joined: 14 Aug 2019
Posts: 1378
Own Kudos [?]: 846 [0]
Given Kudos: 381
Location: Hong Kong
Concentration: Strategy, Marketing
GMAT 1: 650 Q49 V29
GPA: 3.81
Send PM
Re: The proposal to hire ten new police officers in Middletown is quite fo [#permalink]
BoburMirzo26 wrote:
The question stem already mentions that new officers will generate new caseloads of arrests. What happens when they generate more arrests? -> not decrease in crime? All i mean is that it is aready implied that crime level will decrease after hiring new officers, hence E just restates an implied fact.
Besides, the question is quite misleading on cause and effect:
- by OA E it implies that decrease in crime -> decrease in arrests and caseloads. But decreasing crime itself requires arrests and caseloads and will eventually lead to higher imprisonment. It is just a question of time - when will the officers be able to reach minimum crime level

I hope it is not an OG question


Actually it is OG question.

You can understand it with the following example:
Now: Crime rates 1000: Arrests workload etc.: 500 ( say 50% result in extra workload). say Need 50 staff to handle .
argument says: Arrest workload could increase extra 100 arrests and need 10 more staff to handle. Need extra money for this 10 staff.

E. After hiring new police officers: Crime: 600 ; arrests workload :300 ( say 50% result in extra workload )
apply argument in E : Arrest workload could increase extra 100 arrests and need 10 more staff to handle.
It means staff can manage without hiring new employees. No new employees no extra cost. So weakens the conclusion that there would be additional cost to manage extra workload.
GMAT Club Bot
Re: The proposal to hire ten new police officers in Middletown is quite fo [#permalink]
 1   2   
Moderators:
GMAT Club Verbal Expert
6920 posts
GMAT Club Verbal Expert
238 posts
CR Forum Moderator
832 posts

Powered by phpBB © phpBB Group | Emoji artwork provided by EmojiOne